LSAT and Law School Admissions Forum

Get expert LSAT preparation and law school admissions advice from PowerScore Test Preparation.

 Administrator
PowerScore Staff
  • PowerScore Staff
  • Posts: 8916
  • Joined: Feb 02, 2011
|
#38363
Complete Question Explanation
(The complete setup for this game can be found here: lsat/viewtopic.php?t=15015)

The correct answer choice is (C)

This question stem establishes that P is fifth. Since P and S are consecutive, but our Not Laws prohibit S from being fourth, it logically follows that S must be sixth:
PT71_D13 LG Explanations_Game #4_#19_diagram 1.png
If you were able to conclude that S must be sixth, you should quickly scan the five answer choices in the hopes of finding one that agrees with your inference. After all, this is a Must Be True question: attempting to determine the positions of the five remaining variables may be unnecessary if an answer choice reflects the inference already made. We can safely choose answer choice (C) and move on.

Let’s imagine, however, that you arrived at this question without the requisite Not Laws on the fourth position. After all, many students do not see this inference during the setup. In that case, you would need to create two local setups, reflecting the dual nature of the rotating PS block:
PT71_D13 LG Explanations_Game #4_#19_diagram 2.png
Next, you need to accommodate the R _ M split-block, because that block occupies three consecutive positions and provides the highest level of restriction in the game. While this is not a problem in our first setup, it is in the second, as there are only two consecutive positions available at either end of the diagram. Since the second setup does not provide a workable solution to the game, we can safely conclude that the first setup is the only one possible, proving that S must be sixth.

Even though this is a Local question, the conclusion that your second setup does not work has some global ramifications. Clearly, if P is fifth, then S cannot be fourth. But the same would be true if P were third, because a rotating PS block in the third and fourth positions would prevent us from accommodating V. Either way, you can safely assume that S cannot be fourth, and add this Not Law to your main diagram. Further, since P and S are interchangeable and functionally identical (there are no rules that apply to P but not to S, or vice versa), we can draw the same conclusion about P.

If you decided to approach this game with templates, only Templates 1 and 2 allow for P to be fifth. In both templates, S would be forced into the sixth position, immediately proving answer choice (C) correct.
You do not have the required permissions to view the files attached to this post.

Get the most out of your LSAT Prep Plus subscription.

Analyze and track your performance with our Testing and Analytics Package.